You are interested in constructing a 90% confidence interval for the proportion of all caterpillars that eventually become butterflies. Of the 423 randomly selected caterpillars observed, 52 lived to become butterflies. a. With 90% confidence the proportion of all caterpillars that lived to become a butterfly is between and .

Answers

Answer 1
To construct a confidence interval for the proportion of all caterpillars that lived to become a butterfly, we can use the following formula:

Confidence interval = sample proportion ± margin of error

where the sample proportion is the number of caterpillars that lived to become a butterfly divided by the total number of observed caterpillars, and the margin of error is calculated using the following formula:

Margin of error = critical value × standard error

The critical value can be found using a z-table or calculator, and is based on the level of confidence and the degrees of freedom (which is n - 1 for a proportion). For a 90% confidence interval with 422 degrees of freedom, the critical value is approximately 1.645.

The standard error can be calculated using the following formula:

Standard error = sqrt((sample proportion × (1 - sample proportion)) / sample size)

Plugging in the values we have, we get:

Sample proportion = 52 / 423 = 0.123
Standard error = sqrt((0.123 × (1 - 0.123)) / 423) = 0.024

So the margin of error is:

Margin of error = 1.645 × 0.024 = 0.039

Therefore, the 90% confidence interval for the proportion of all caterpillars that lived to become a butterfly is:

0.123 - 0.039 < p < 0.123 + 0.039

0.084 < p < 0.162

So with 90% confidence, the proportion of all caterpillars that lived to become a butterfly is between 0.084 and 0.162.

Therefore, the answer is between 0.084 and 0.162.

Related Questions

Solve -8x = 56 for x.
Ox=6
Ox= -7
Ox=7
Ox= -8

Answers

-7.

Explanation: 8 is multiplying with X. Therefore you must do the opposite to the answer. 56/-8= -7.

Answer:

-7

Step-by-step explanation:

you divide -8 on both sides

the two -8 on your left will cancel each other

and 56 divided by-8=-7

Find the x intercepts of the parabola with vertex (-1,2) and y intercept (0,3).

Answers

The x intercepts of the parabola with  vertex (-1,2) and y intercept (0,3) does not exist

Finding the x intercepts of the parabola

From the question, we have the following parameters that can be used in our computation:

vertex (-1,2) and y intercept (0,3).

A parabola is represented as

y = a(x - h)² + k

Where

Vertex = (h, k)

Using the above as a guide, we have the following:

y = a(x + 1)² + 2

Also, we have

a(0 + 1)² + 2 = 3

So, we have

a = 1

This means that

y = (x + 1)² + 2

Set to 0 to calculate the x-intercepts

(x + 1)² + 2 = 0

This gives

(x + 1)² = -2

Take the square root

(x + 1) = comple numbers

Hence, the x intercepts of the parabola does not exist

Read more about x intercepts at

https://brainly.com/question/3951754

#SPJ1

I NEED HELP PLEASEEE

Answers

A description that best compare the graph of the functions is: B. the line for function A is steeper.

How to determine an equation of this line?

In Mathematics and Geometry, the point-slope form of a straight line can be calculated by using the following mathematical equation:

y - y₁ = m(x - x₁)

Where:

x and y represent the data points.m represent the slope.

First of all, we would find the slope of function B;

Slope (m) = (y₂ - y₁)/(x₂ - x₁)

Slope (m) = (-1 + 2)/(3 - 0)

Slope (m) B = 1/3.

At data point (0, -1) and a slope of 1/3, function B can be calculated by using the point-slope form as follows:

y - y₁ = m(x - x₁)

y + 1 = 1/3(x - 0)

y = 1/3(x) - 1

In conclusion, the line for function A is steeper because it has a greater slope than function B i.e 3 > 1/3.

Read more on slope and function here: https://brainly.com/question/10344919

#SPJ1

Pls answer this quickly. I need at atleast 70%

Answers

The equation of the line with yellow point in slope intercept form is y = 7 / 6 x + 2.

How to find the equation of a line?

The equation of a straight line can be represented in different form such as slope intercept form, point slope form, etc.

Therefore, let's represent the line with yellow point in slope intercept form.

Hence,

y = mx + b

where

m = slopeb = y-intercept

Therefore, using (0, 2) and (6, 9).

m = 9 - 2 / 6 - 0

m = 7 / 6

Hence, let's find the y-intercept

y = 7 / 6 x + b

2 = 7 / 6(0) + b

b = 2

Therefore,

y = 7 / 6 x + 2.

learn more on slope intercept form here:https://brainly.com/question/29146348

#SPJ1

Express sin L as a fraction in simplest terms

Answers

The SinL as a fraction in the simplest form is 12/13.

We are given

The side NM = 5,

The side LM = 12,

By using Pythagoras' theorem, we can say that;

NL² = NM² + LM²

Substitute the values of NM and LM;

NL² = 12² + 5²

NL = 13,

For Sin L = perpendicular side/hypotenuse side

Sin L = NM / NL,

Sin L = 12/13

Therefore, the SinL as a fraction in the simplest form is 12/13

To know more about the right-angled triangle:

brainly.com/question/3770177

#SPJ1

The table shows how many Calories Lance burns jogging on a treadmill for different lengths of time. Lance's goal today is to burn 380 Calories. If he has been jogging for 25.5 minutes, for how many more minutes does he still need to jog? Assume the situation is proportional.

Answers

Lance needs to jog for an additional 316.08-25.5 = 290.58  minutes to burn 380 Calories.          

The information given in the table to  produce a direct equation that relates the number of Calories burned to the time spent jogging. Let's use the  pitch- intercept form y =  mx b, where y represents the number of Calories burned( in hundreds), x represents the time spent jogging( in  twinkles), m represents the rate of Calories burned( in hundreds per  nanosecond), and b represents the  original Calories burned( in hundreds).  To find m, we can use the points( 10, 13) and( 15, 19), which represent two ordered  dyads on the line. We can find the  pitch of the line using the  pitch formula  

m = ( y2- y1)/( x2- x1)  m = ( 19- 13)/( 15- 10)  

m = 1.2  So the equation for the line is  y = 1.2 x b.

 To find b, we can use the point

( 10, 13)  = 1.2( 10) b  b = 0.7

 So the complete equation for the line is  y = 1.2 x0.7  Now we can use this equation to  break for x, the number of  twinkles Lance needs to jog to burn 380 Calories.  

380 = 1.2 x0.7  = 1.2 x  x = 316.08

So Lance needs to jog for an additional 316.08-25.5 = 290.58  minutes to burn 380 Calories.

To know more about Additional .

https://brainly.com/question/4721701

#SPJ11

Ju used 6
bags of mulch in her front yard and
2 bags of mulch in her backyard. How many
bags of mulch has Ju used (in simplest form)?

Answers

Answer:

she used 8 bags of mulch altogether.

Step-by-step explanation:

if this isn't correct, you may want to reword or specify the question.

In 2017 the population of Rexburg, Idaho was 28,337 people. The population was expected to grow at a rate of about 1.55% per year. Based on these numbers, what would we predict the population of Rexburg will be in the year 2022?

(Round to the nearest whole number.)

Number
people

Answers

Answer: 30,620 people

Step-by-step explanation:

We will use the given formula for exponential growth.

➜ 1.55% / 100 = 0.0155

➜ 2022 - 2017 = 5 years

          [tex]A=Pe^{rt}[/tex]

          [tex]A=(28,337\;people)e^{(0.0155)(5\;years)}[/tex]

          [tex]A=(28,337\;people)e^{0.0775}[/tex]

          [tex]A=(28,337\;people)e^{0.0775}[/tex]

          [tex]A=30,620.4587 \approx 30,620\;people[/tex]

I need help graphing 2x-5y=10

Answers

Answer:

Try the desmos graphing calculator:

Step-by-step explanation:

Please calculate this :

Answers

The value of the expression is determined as 14.

What is the value of the expression?

The value of the expression is calculated as follows;

We will apply the principle known as BODMAS;

B - bracket

O - Off

D - division

M - multiplication

A - addition

S - subtraction

The given expression; = (27 · 3  -  1719 ÷ 1719) ÷ 8 + 5² - 3³  + (8 · 4  -  2·13)

We will simplify the expression as follows;

(27 · 3  -  1719 ÷ 1719) = (81  -  1719/1719) = (81 - 1) = 80

(8 · 4  -  2·13) = (32  -  26) = 6

= 80 ÷ 8 + 5² - 3³  + 6

= (80/8) +  5² - 3³  + 6

= 10 +  5² - 3³  + 6

= (10 + 5² + 6) - 3³

= 41 - 27

= 14

Learn more about BODMAS here: https://brainly.com/question/27985272

#SPJ1

The scatter plot shows the weights and age of a puppy and a good line of fit. The equation for the line of fit shown is:
y = 2x + 1.5
Where x represents the puppy’s age in weeks and y represents the puppy’s weight. Predict the weight of a 12-week-old puppy in pounds. Show your work.

Answers

The predicted weight of a 12-week-old puppy is 25.5 pounds.

To predict the weight of a 12-week-old puppy using the line of fit equation y = 2x + 1.5, follow these steps:
1. Identify the value of x: In this case, x represents the puppy's age in weeks, so x = 12.
2. Plug the value of x into the equation: y = 2(12) + 1.5.
3. Solve the equation: y = 24 + 1.5.
4. Calculate the result: y = 25.5.
So, the predicted weight of a 12-week-old puppy is 25.5 pounds.

For more questions on weight

https://brainly.com/question/28218377

#SPJ11

Using the best of fit equation given , the predicted weight of a 12 - weeks old puppy would be 25.5lb

Line of fit equation

The line of fit is used to establish a mathematical relationship between two variables. A good line of fit would give a good mathematical relationship between related variables.

In the question above; our line of fit equation is y = 2x + 1.5

x = puppy's age ; y = weight

To obtain the value of y for any value of x ; we substitute either of the given value .

For x = 12

y = 2(12 ) + 1.5

y = 24 + 1.5

y = 25.5

Hence, weight of puppy after 12 weeks will be 25.5lbs

Learn more on line of best fit ; https://brainly.com/question/17013321

#SPJ1

Given f(x) = 3x² + 9x - 16, find f(-8)

Answers

Answer:

To find f(-8), we need to substitute -8 for x in the expression for f(x) and evaluate:

f(-8) = 3(-8)² + 9(-8) - 16

Simplifying this expression, we get:

f(-8) = 3(64) - 72 - 16

f(-8) = 192 - 72 - 16

f(-8) = 104

Therefore, f(-8) = 104 when f(x) = 3x² + 9x - 16.

Learn more about Graphing Functions here:

https://brainly.com/question/23945486

#SPJ5

F(-8)=104 is the answer given 3x^2 + 9x - 16

!! Will give brainlist !!
(Need help asap!!)

Determine the measure of each arc.

Answers

The length of the arc are

Arc MN =  72 degrees

Arc NQR = 180 degrees

Arc NQ = 108 degrees

Arc MRP =  193 degrees

Arc QR = 72 degrees

Arc QMR = 288 degrees

Arc PQ = 13 degrees

Arc PRN = 265 degrees

Arc MQN = 288 degrees

How to find the length of each arc

The relationship between central angle and intercepted arc is that they are equal. This is used in solving the length of arcs

hence we have that

angle ROQ = angle NOM =  72 degrees (vertical angles)

Arc MN = angle NOM =  72 degrees

Arc NQR = 180 degrees (semicircle)

Arc NQ

angle POQ = 180 - 72 - 95

angle POQ = 13

Arc NQ = 95 + 13 = 108 degrees

Arc MRP = 180 + 13 = 193 degrees

Arc QR = 72 degrees

Arc QMR = 360 - 72 = 288 degrees

Arc PQ = 13 degrees

Arc PRN = 360 - 95 = 265 degrees

Arc MQN = 360 - 72 = 288 degrees

Learn more about intercepted arc at

https://brainly.com/question/15899344

#SPJ1

please help me with my geometry i cant fail this class or i’ll have to move out of my parents house :(

Answers

The required answer is mAD = 126°

How did we get the value?

Step 1: Recall Inscribed Angles of a Circle Theorem.

Step 2: It means m∠ABD ≡ m∠ACD. So, it can be written:

(11x - 3)° (8x + 15)°

Step 3: Evaluate for x:

11x - 3 = 8x + 15

X = 6

Step 4: Substitute x = 6 in 11x - 3 as:

11(6) - 3

Step 5: Evaluate for answer:

11 × 6 - 3

= 63

Step 6: Now recall Measure of an Inscribed Angle:

m∠ABD = ¹/₂ MAD

Step 7: Substitute m∠ABD = 63 in above equation:

63 = ½ mAD

Step 8: Evaluate for answer:

63 = ¹/₂ AD

AD = 126

Step 9

Thus, the required answer is:

MAD = 126°

Solution

The required answer is:

mAD = 126°

learn more about angles of a circle theorem: https://brainly.com/question/26403793

#SPJ1


Express the area of the entire rectangle.
Your answer should be a polynomial in standard form.

Answers

The value of area of rectangle is,

⇒ Area = x² + 10x + 21

We have to given that;

Lenght = (x + 7)

Width = (x + 3)

Hence, We can formulate;

The value of area of rectangle is,

⇒ Area = Lenght x width

⇒ Area = (x + 7) (x + 3)

⇒ Area = x² + 7x + 3x + 21

⇒ Area = x² + 10x + 21

Thus, The value of area of rectangle is,

⇒ Area = x² + 10x + 21

Learn more about the rectangle visit:

https://brainly.com/question/2607596

#SPJ1

A school is gathering some data on its sports teams because it was believed that the distribution of boys and girls were evenly distributed across all the sports. This table lists the number of boys and girls participating in each sport.

Boys Girls
Tennis 18 30
Soccer 42 15
Swimming 12 18

Select the observed and expected frequencies for the boys participating in soccer.

a) Observed: 42 Expected: 22.5

B) Observed: 42 Expected: 24

C) Observed: 57 Expected: 24

D) Observed: 57 Expected: 22.5

Answers

The observed frequency = 42, and the expected is 22.5.

option A.

What is the observed and expected frequencies?

The observed and expected frequencies for the boys participating in soccer is calculated as follows;

Total boys = 18 + 42 + 12 = 72

Total girls = 30 + 15 + 18 = 63

Total number of the students = 72 + 63 = 135

Ratio of boys = 72/135 = 0.533

Ratio of girls = 63 / 135 = 0.467

The observed and expected frequencies for the boys participating in soccer is calculated as follows;

Expected frequency = 0.534 x 42

= 22.4

Thus, the observed frequency = 42, and the expected is 22.5.

Learn  more about observed frequency here: https://brainly.com/question/23866673

#SPJ1

Answer:

Observed: 42

Expected: 24

Step-by-step explanation:

If we simply go to the chart then we can directly see the observed frequency for boys participating in soccer is 42.

To find the expected frequency, we need to find the number of occurrences if the null hypothesis is true, which in this case, was that the three options are equally likely, or if the three options were all evenly distributed.

First, add up all the options in the boys column:

18 plus 42 plus 12 equals 72

If each of these three options were evenly distributed among the 72 boys, we would need to divide the total evenly between the three options:

72 divided by 3 equals 24

This means we would expect 24 boys to choose tennis, 24 boys to choose soccer, and 24 boys to choose swimming.

What is the surface area of the triangular prism?
S.A.=blank ft/2
4ft,3ft,2ft,5ft
PLEASE HELP!
I NEED IT FOR MONDAY BEFORE MIDNIGHT
I’LL GIVE BRAINLEST
LEFEL F NET AND SURFACE AREA

Answers

The surface area of the triangular prism is 84 ft/2

How to solve

The surface area of a triangular prism is found with: Area = bh + (s1 + s2 + s3)L.

Given a 4ft base, 3ft height, 5ft slant height, and 2ft length, we substitute to get:

Area = (4ft3ft) + 3(5ft*2ft) =

[tex]12ft^2 + 30ft^2 = 42ft^2.[/tex]

In the specified format, the surface area of the triangular prism is 84 ft/2.

This is because the format given is blank ft/2

learn more surface area of prism fromhttps://brainly.com/question/1297098

#SPJ1

Could you provide the formula to calculate the surface area of a triangular prism, given dimensions of 4ft, 3ft, 2ft, and 5ft? The surface area should be presented as (blank ft/2).

What are the answer these questions?

Answers

The right Riemann sum is an overestimate.

The left Riemann sum is an underestimate.

For n=2, the interval [0,1] is divided into 2 subintervals of equal width, so Δx = (1-0)/2 = 1/2.

Left Riemann Sum with n=2:

The left endpoints of the subintervals are x=0 and x=1/2.

f(0) = e⁰ = 1

f(1/2) = [tex]e^1^/^2[/tex]

So, the left Riemann sum is:

Δx[f(0) + f(1/2)] = (1/2)[1 +[tex]e^1^/^2[/tex]] = (1/2) + (1/2)[tex]e^1^/^2[/tex]

sum = (1/2) + (1/2)[tex]e^1^/^2[/tex]

Right Riemann Sum with n=2:

The right endpoints of the subintervals are x=1/2 and x=1.

f(1/2) = [tex]e^1^/^2[/tex]

f(1) = e¹ = e

So, the right Riemann sum is:

Δx[f(1/2) + f(1)] = (1/2)[[tex]e^1^/^2[/tex] + e] = (1/2)[tex]e^1^/^2[/tex] + (1/2)e

sum = (1/2)[tex]e^1^/^2[/tex]+ (1/2)e

Since [tex]e^1^/^2[/tex] > 1, the right Riemann sum is an overestimate, and the left Riemann sum is an underestimate.

To learn more on Equation:

https://brainly.com/question/10413253

#SPJ1

While browsing at her local farmers' market, Lexi stops at a booth selling fancy candles and hand soaps. She has
$50 to spend. Candles cost $12.50 each, and hand soaps cost $5 each.
Graph the inequality that represents how many candles, x, and hand soaps, y, Lexi can buy.
Plot points on the boundary line. Select the line to switch between solid and dotted. Select a region to shade it.

Answers

The graph represents the valid region of the number of candles and hand soaps that Lexi can buy with her $50 budget.

To shade the region that satisfies the inequality, we need to determine which side of the line represents the valid region. Since the inequality is less than or equal to, the valid region is the one below the line.

Let x be the number of candles that Lexi buys and y be the number of hand soaps that she buys. The cost of x candles is 12.50x and the cost of y hand soaps is 5y. Lexi has $50 to spend, so we can write the following inequality:

12.50x + 5y ≤ 50

To graph this inequality, we first need to graph the boundary line, which is the equation obtained by replacing the inequality sign with an equal sign:

12.50x + 5y = 50

We can rewrite this equation in slope-intercept form as:

y = -2.50x + 10

To plot points on the boundary line, we can choose any two values of x and solve for the corresponding values of y. For example, if we let x = 0, then y = 10, so one point on the line is (0,10). If we let x = 4, then y = 0, so another point on the line is (4,0).

We can now graph the boundary line, which has a slope of -2.50 (meaning it goes down and to the right) and a y-intercept of 10 (meaning it intersects the y-axis at the point (0,10)). To switch between a solid and dotted line, select the "solid line" or "dotted line" option in the graphing tool.

For such more questions on Inequality:

https://brainly.com/question/25275758

#SPJ11

What's the probability of rolling a prime number on a fair six-sided die?

Answers

Answer:

66.666%

Step-by-step explanation:

the numbers on the dice would be 1,2,3,4,5,6

1,2,3,5 are all prime numbers.

4/6 =2/3 =66.666%

Answer

the probability is 3 / 6 and that simplifies to 1 / 2 or 0.5.

Explanation

The primes from a six-sided die are 2, 3, 5. 4 and 6 are composite, and 1 is not defined either way,

Multiplication What 7 x 4 / 8

Answers

Answer:

7/2

Step-by-step explanation:

7 x 4 / 8

= 7 x 1/2

= (7 x 1)/2

= 7/2

Note

1. When you see a fraction, first simplify the numerator and denominator.

That is,

In 4/8,

4 is the numerator

8 is the denominator

4/8 = 1/2

2. Multiply number with numerator, then divide it with the denominator

7 x 1/2

7 x 1 = 7

(7 x 1)/2 = 7/2

What is the equation of the line that passes through the points given in the table x -3, 3, 6 y 3, 5, 6,
A y= -3x - 6
B y = -1/3x + 8
C y= 1/3x + 4
D y= 3x - 12

Answers

Answer:

C. y = 1/3x + 4

Step-by-step explanation:

To find the equation of the line that passes through the given points, we can use the slope-intercept form of the equation of a line, which is:

y = mx + b

where m is the slope of the line and b is the y-intercept.

To find the slope of the line, we can use the formula:

m = (y2 - y1) / (x2 - x1)

where (x1, y1) and (x2, y2) are any two points on the line. We can choose the points (x1,y1) = (-3,3) and (x2,y2) = (6,6) from the table.

m = (6 - 3) / (6 - (-3)) = 3/9 = 1/3

Now that we have the slope, we can use the point-slope form of the equation of a line to find the y-intercept:

y - y1 = m(x - x1)

where (x1,y1) is any point on the line. We can choose the point (x1,y1) = (3,5) from the table.

y - 5 = (1/3)(x - 3)

Now we can solve for y to get the equation in slope-intercept form:

y = (1/3)x + (5 - 1/3*3)

y = (1/3)x + 4

Therefore, the equation of the line that passes through the given points is C) y= 1/3x + 4.

Rewrite the equation 5(x – 3) = x + 13 with 6 substituted for x. Write a question mark over the equal sign to show that you're testing to see if the equation is true. edmentum math practice. PLEASE HELP ME.

Answers

Answer:

Step-by-step explanation:

5(x - 3) ? x + 13

if x = 6

5(6 - 3) ? 6 + 13

15 ? 19

15 ≠ 19

Step-by-step explanation:

When substituting 6 for x, the equation becomes:

5(6 - 3) ?= 6 + 13

Now, we can solve both sides of the equation:

5(3) ?= 6 + 13

15 ?= 19

Since these are not equal, the equation is not true for x = 6.

100 Points! Algebra question. Looking for an answer to A and B. Photo attached. Please show as much work as possible. Thank you!

Answers

A) A above middle C with a frequency of 440 cycles per second is 49 notes up the piano keyboard.

B) frequency of the pitch that is 73 notes up the keyboard is 1760 cycles per second.

How to determine frequency?

A. To find how many notes up the piano keyboard the pitch with a frequency of 440 cycles per second is, solve the formula

n = 1 + 12 × log₂(f/27.5) for n,

where f = frequency of the pitch to find and n = number of notes up the keyboard.

Substituting f = 440:

n = 1 + 12 × log_2(440/27.5)

n = 1 + 12 × log_2(16)

n = 1 + 12 × 4

n = 49

Therefore, the A above middle C with a frequency of 440 cycles per second is 49 notes up the piano keyboard.

B. Using the same formula, n = 1 + 12 × log₂(f/27.5), to find the frequency of the pitch that is 73 notes up the keyboard. Solving for f:

73 = 1 + 12 × log₂(f/27.5)

72 = 12 × log₂(f/27.5)

6 = log₂(f/27.5)

2⁶ = f/27.5

f = 27.5 x 2⁶

Therefore, the frequency of the pitch that is 73 notes up the keyboard is:

f = 27.5 x 2⁶ = 1760 cycles per second.

Find out more on frequency here: https://brainly.com/question/254161

#SPJ1

If m d=18 and m c = 45 what is m bc

Answers

The measure of angle BC is 27 degrees.

To find the measure of angle BC, we need to use the angle sum property of triangles. In triangle ABC, the sum of all angles is equal to 180 degrees. We know that angle ACD is a straight angle and therefore measures 180 - 45 = 135 degrees.

Additionally, we know that angle ADB is also a straight angle and therefore measures 180 - 18 = 162 degrees.

To find angle BCD, we subtract the measure of angle ACD from the measure of angle ADB, which gives us:

m BCD = m ADB - m ACD
m BCD = 162 - 135
m BCD = 27 degrees

To learn more about : angle

https://brainly.com/question/25770607

#SPJ11

A plumber charges fixed and per hour. He did a job for a neighbor for 3 hours, he charged $140. To another
neighbor the plumber charged him $232 for 7 hours.

1.Find out how much he charges per hour
2. how much fixed rate
3. the equation that tells how much the plumber earns.
4. how much would you earn if you worked 5.75 hours

Answers

The plumber charges $23 per hour, the plumber charges a fixed rate of $71 and If someone worked for 5.75 hours, their earnings would be $203.25

To find the plumber's hourly rate, we can use the given information and set up two equations:

3h + f = 140 (where h is the hourly rate and f is the fixed rate for the first job)

7h + f = 232 (where h is the hourly rate and f is the fixed rate for the second job)

We can solve for h by subtracting the first equation from the second equation:

4h = 92

h = 23

Therefore, the plumber charges $23 per hour.

To find the plumber's fixed rate, we can substitute h = 23 into one of the equations and solve for f:

3h + f = 140

3(23) + f = 140

69 + f = 140

f = 71

Therefore, the plumber charges a fixed rate of $71.

The equation that tells how much the plumber earns can be written as:

Earnings = (hourly rate x number of hours) + fixed rate

or E = 23h + 71

If someone worked for 5.75 hours, their earnings would be:

Earnings = (hourly rate x number of hours) + fixed rate

Earnings = (23 x 5.75) + 71

Earnings = 132.25 + 71

Earnings = $203.25

To learn more on Equation:

https://brainly.com/question/10413253

#SPJ1

Point C is located at the origin.

Answers

a The slope of line £ is 3/4.

b The equation for £ is y = 3/4x + 3.

c The radius of OC is 5.

d The distance from £ to OC along the y-axis is 3.

How to calculate the value

The slope of a line is the change in y over the change in x. In this case, the change in y is 3 and the change in x is 4. So, the slope of line £ is 3/4.

The equation for a line can be written in the form y = mx + b, where m is the slope and b is the y-intercept. In this case, the slope is 3/4 and the y-intercept is 3. So, the equation for £ is y = 3/4x + 3.

The radius of a circle is the distance from the center to any point on the circle. In this case, the center of the circle is at the origin, which is at (0, 0). The point on the circle is at (-4, 3). So, the radius of the circle is the distance from (0, 0) to (-4, 3).

This distance can be found using the Pythagorean theorem:

r² = 4² + 3²

r² = 25

r = 5.

Leaen more about slope on

https://brainly.com/question/3493733

#SPJ1

A sporting goods store records the sales of gloves over a 10-day period. They also record each day’s low temperature. The scatter plot shows the data and a good line of fit. The equation for the line of fit shown is y = -1.2x + 122. What is the y-intercept of the equation, and what does it mean?

Answers

The y- intercept represents the anticipated number of gloves  vended on a day when the low temperature is 0 degrees.    

The terms you mentioned are  formerly included in the problem description.  The y- intercept of the equation is the value of y when x =  0. In the given line of fit equation,

y = -1.2 x 122, the y- intercept is 122.  The y- intercept represents the anticipated number of gloves  vended on a day when the low temperature is 0 degrees. In this case, when the low temperature is 0,

the store is  prognosticated to  vend 122 gloves. Keep in mind that this is an estimate grounded on the line of fit and may not be an exact value.

For more questions on y-intercept

https://brainly.com/question/30886022

#SPJ11

express 2/5-4/y+3 as a single fraction

Answers

Answer:

To express the expression 2/5 - 4/y + 3 as a single fraction, we need to find a common denominator for the fractions involved.

The common denominator for 5 and y is 5y.

First, we'll rewrite 2/5 as an equivalent fraction with the denominator 5y:

2/5 = (2 * y)/(5 * y) = 2y/5y

Next, we'll rewrite 4/y as an equivalent fraction with the denominator 5y:

4/y = (4 * 5)/(y * 5) = 20/5y

Now, we can rewrite the expression with the common denominator 5y:

2y/5y - 20/5y + 3

Since the denominators are now the same, we can combine the numerators:

(2y - 20 + 3 * 5y)/(5y)

Simplifying further:

(2y - 20 + 15y)/(5y) = (17y - 20)/(5y)

Thus, the expression 2/5 - 4/y + 3 can be expressed as a single fraction: (17y - 20)/(5y).

What is the probability that both events occur?

Answers

Answer:

P(A and B) = P(A)P(B)

= (1/3)(1/2) = 1/6

= about .17 = about 17%

1/6, because 1/3 (1 and 6 out of 1, 2, 3, 4, 5, and 6) x 1/2 (2, 4, 6 out of 1, 2, 3, 4, 5, 6)
Other Questions
the coriolis deflection is negligible in a tornado because question 15 options: tornadoes originate over the equator. the system is too small to be significantly affected by the earth's rotation. of surface roughness. all of the answers are correct. none of the answers is correct. Find the volume of liquid needed to fill a sphere of radius R to height R/5. Which diagram best represents how geography affected an Africancivilization?OA.B.C.D.Some African climatesare poor for farming.Water levels rise inthe Red Sea.Random flooding occursin the Niger River.Rain forests spread inCentral Africa.[40The Bantu migrationbegins.The Kingdom ofAxum collapses.AThe Songhai peoplemust live as nomads.The Kingdom of Malibecomes wealthy. what is a symptom of anemia in which spongy bone invades the eye sockets? ECT is a neural stimulation technique that is most commonly used on which type of patients? Evaluate the indefinite integral. (Use C for the constant of integration. Remember to use absolute values where appropriate.) st tan(2x + 9)dx a product labeled "new and improved" would fall within the category of a Which of the following is an example of a variable cost for an amusement park?A) salary of the park managerB) food cart suppliesC) liability insuranceD) interest on the property's mortgageE) property taxes how do the landscape paintings of john constable evoke themes of romanticism in art? . Francois has 8 feet of trim. He ismaking 3 picture frames. Write afraction that represents the number offeet of trim that can be used for eachpicture frame. Fill in the bubble beforeeach number that is correct.Pls help its a grade 4.Look at the picture below.What does the diagram show?A. refractionB. absorptionC. reflectionD. diffraction (x - 2)/(x + 2) - (x + 2)/(x - 2) Which utilities can be used to manage printers? (Choose all that apply.)a. the Devices and Printers appletb. Computer Managementc. Device Managerd. Print Management snap-in With the above statements in mind, make a research project on the cost and revenue of a Bricklaying business in your area. The aim of a business is produce Bricks at lower cost per unit and to create more jobs to the local people around the area. 1 Write a short report (3) a cause of female infertility is failure of ovulation due to hyposecretion of hormones from the Which of the following is not an example of a mnemonic technique that emphasizes organization?a. Keyword methodb. Hierarchyc. First-letter techniqued. Narrative technique A golfer played 7 rounds of golf with the following scores: 1 round at 3 over par, 2 rounds with 1 over par, 3 rounds with 3 under par, and the last round at 1 under par. How did she finish with respect to par? The ability to choose a pricing policy and apply it to your product or service is an important skill for all entrepreneurs. This activity will provide practice doing basic pricing calculations, using your product costs and your markup.Step 1: Identify Pricing Risk FactorsDetermine risks associated with your pricing objectives. Look at how the price is affected by:supply and demandthe competitioneconomic conditionsgovernment regulationsHow does the target market itself affect your pricing?Open a document and title it Pricing.Create a heading on the document and label it Risk Factors.Write four to five sentences that describe your product in relation to each of the specific risk factors.Step 2: Determine Your Pricing Objective(s)Determine your pricing objective. Note that you can choose to maximize sales, profits, market share, or simply to cover your costs and earn a modest profit.Create a second heading on the document and label it Pricing Objectives.Write your pricing objective in one or two sentences. Note that you may decide to optimize on more than one objective at the same time. Tommy and his friends are planning a trip to the Mega Vault trampoline park. The employeeTommy spoke to on the phone said the total cost for all 5 of them would be $85. This includesa $3 fee added to each entrance ticket to cover the cost of a pair of grip socks.Which equation can you use to find c, the cost of an entrance ticket?5(c + 3) = 855c + 3 - 853c+5 853(c+5)= 85 An injection of ACTH would cause an increase in the release of which of the following hormones?A) thyroxinB) glucocorticoidsC) growth hormoneD) antidiuretic hormone